1 Problems in Round 2
√
1. Given a, b, c ≥ 0 satisfying a + b + c = 5, find the maximum value
of the following expression:
P = (a2 − b2 )(b2 − c2 )(c2 − a2 ).
Solution
WLOG assume c ≥ b ≥ a; so that we have
(a2 − b2 )(b2 − c2 )(c2 − a2 ) = (c2 − b2 )(c2 − a2 )(b2 − a2 ) ≤ b2 c2 (c2 − b2 ).
√
Also note that 5 = a + b + c ≥ b + c since a ≥ 0.
Now, using the AM-GM inequality we have
√ ! !2 √ ! !2
5 5
(c + b) · −1 ·c · + 1 b · (c − b)
2 2
(√ )5
5(b + c) √
≤ (c + b) ≤ 5;
5
So that we get √
P ≤ 5.
√ √ !
5 5
And hence we are done. Equality holds if and only if (a, b, c) = + 1; − 1; 0
2 2
and its cyclics.
2. a, b, c > 0. Show that we have
r
ab + bc + ca + 1 3 3 (a + b)(b + c)(c + a)
+ ≥ 1.
(a + b + c + 1)2 8 abc
First Solution
Using AM-GM on the four terms in the left side, we have
r s
ab + bc + ca + 1 3 3 (a + b)(b + c)(c + a) (ab + bc + ca + 1)(a + b)(b + c)(c + a)
2
+ ≥44 .
(a + b + c + 1) 8 abc 83 abc(a + b + c + 1)2
Hence it is sufficient to check that
(ab + bc + ca + 1)(a + b)(b + c)(c + a) ≥ 2abc(a + b + c + 1)2 .
Multiplying both sides with 2; we are required to prove that
! !
X X
c(a + b) + 2 ab(a + b) + 2abc ≥ 4abc(a + b + c + 1)2 .
cyc cyc
1
In this form, it is a direct application of Cauchy-Schwarz inequality, since
! !
X X √ √ 2
c(a + b) + 2 ab(a + b) + 2abc ≥ 2 abc(a + b + c) + 2 abc
cyc cyc
= 4abc(a + b + c + 1)2 .
Second Solution
We prove the inequality
(ab + bc + ca + 1)(a + b)(b + c)(c + a) ≥ 2abc(a + b + c + 1)2 .
Introducing the notations p = a + b + c; q = ab + bc + ca; r = abc we see that
our last inequality equivalents
(q + 1)(pq − r) ≥ 2r(p + 1)2 ;
Or
pq 2 + pq ≥ 3r + 2p2 r + 4pr + qr;
Which, after simple calculations, is equivalent to with
2 1 1 2
p q 2 − 3pr + (pq − 9r) + q (pq − 9r) + p q 2 + 3q − 18r ≥ 0.
3 3 9 9
But, using the obvious facts that pq ≥ 9r and q 2 ≥ 3pr; it is enough to prove
that
q 2 + 3q − 18r ≥ 0
Note that from AM- GM inequality we have
p √
q 2 + 3q ≥ 2 3q 3 ≥ 2 3 · 27r2 = 18r;
Because using AM-GM
q 3 = (ab + bc + ca)3 ≥ 27a2 b2 c2 = 27r2 .
Hence proved. Equality holds if and only if a = b = c = 1.
3. For a, b, c > 0 ∧ n ∈ {N − {1}} such that an + bn + cn = 3, prove
that
an+1 · b + bn+1 · c + cn+1 · a ≤ 3.
Solution
Letting x = an ; y = bn ; z = cn we see that the given inequality is equivalent to
√ √ √
x · n xy + y · n yz + z · n zx ≤ 3;
Lemma (V. Cirtoaje)
For a, b, c > 0; ∧a + b + c = 3 we have
√ √ √
a ab + b bc + c ca ≤ 3;
2
proof
We rewrite the inequality as
√ √ √
3 a ab + b bc + c ca ≤ (a + b + c)2 ;
Or equivalently:
X √ √ √ 2
a − 2 ab + bc − c + ca ≥ 0;
cyc
Which is perfectly true. Hence our lemma is proved.
Note that using Hölder’s inequality we have
√ √ √ n 3 1 3 1 3 1
2
x · n xy + y · n yz + z · n zx ≤ (x + y + z)n−2 · x 2 y 2 + y 2 z 2 + z 2 x 2 .
3 1 3 1 3 1
2
≤ 3n−2 · x 2 y 2 + y 2 z 2 + z 2 x 2
≤ 3n ;
Using the lemma for x, y, z.
And hence we are done. Equality holds if and only if a = b = c = 1.
Remark (Nguyen Huy Tung)
Let a, b, c > 0 satisfy an + bn + cn = 3, and n ≥ 2 is a natural number. Then
we also have the following inequality:
an+1 bn + bn+1 cn + cn+1 an ≤ 3.
4. Given positives a, b, c, show that
a3 + b3 + c3 ab + bc + ca ab bc ca
· 2
≥ 2
+ 2
+ .
abc (a + b + c) ab + 2c bc + 2a ca + 2b2
Solution
We can rewrite the original inequality into:
a3 + b3 + c3 ab + bc + ca X c2
· 2
≥3−2 .
abc (a + b + c) cyc
ab + 2c2
Or;
a3 + b3 + c3 ab + bc + ca X c2
· + 2 ≥ 3.
abc (a + b + c)2 cyc
ab + 2c2
But, from AM-GM and Cauchy-Schwarz inequality we have
a3 + b3 + c3 ab + bc + ca X c2
· + 2
abc (a + b + c)2 cyc
ab + 2c2
a3 + b3 + c3 ab + bc + ca (a + b + c)2
≥ · + 2 ·
abc (a + b + c)2 (ab + bc + ca) + 2(a2 + b2 + c2 )
s
(a3 + b3 + c3 )(ab + bc + ca)(a + b + c)4
≥33 2;
abc(a + b + c)2 (ab + bc + ca + 2a2 + 2b2 + 2c2 )
3
And therefore it is enough to check that
2
(a3 + b3 + c3 )(ab + bc + ca)(a + b + c)2 ≥ abc ab + bc + ca + 2a2 + 2b2 + 2c2 ;
Which, on using the known inequality which can be proved by AM-GM :
(a + b + c)(ab + bc + ca) ≥ 9abc;
Reduces to
2
9(a + b + c)(a3 + b3 + c3 ) ≥ ab + bc + ca + 2a2 + 2b2 + 2c2 .
But, using the Cauchy-Schwarz inequality we have
2
9(a + b + c)(a3 + b3 + c3 ) ≥ 9(a2 + b2 + c2 )2 = 3a2 + 3b2 + 3c2
2
≥ ab + bc + ca + 2a2 + 2b2 + 2c2 ;
And hence we are done.
5. If a, b, c > 0 satisfy c(a − b) 6= 0; Prove that
a3 b3 c3 8(a + b)3 (b + c)3 (c + a)3
+ + + ≥ 4.
(b + c)3 (b + c)3 (c + a)3 (ac − bc)2 (a + b + c)5
Solution
We rewrite our original inequality as
a3 + 2(b + c)3 b3 + (c + a)3 c3 + (a + b)3 8(a + b)3 (b + c)3 (c + a)3
+ + + ≥ 8.
(b + c)3 (c + a)3 (a + b)3 (ac − bc)2 (a + b + c)5
On directly applying AM-GM we get
a3 + 2(b + c)3 b3 + (c + a)3 c3 + (a + b)3 8(a + b)3 (b + c)3 (c + a)3
+ + +
(b + c)3 (c + a)3 (a + b)3 (ac − bc)2 (a + b + c)5
s
8 {a3 + 2(b + c)3 } {b3 + (c + a)3 } {c3 + (a + b)3 }
≥44 ≥ 8;
(ac − bc)2 (a + b + c)5
⇐⇒ a3 + 2(b + c)3 b3 + (c + a)3 c3 + (a + b)3 ≥ 2(ac − bc)2 (a + b + c)5 .
But, applying Hölder’s inequality we have
(a + b + c)6
b3 + (c + a)3
3
c + (a + b)3 ≥
.
16
And hence we are only required to prove that
3
a + 2(b + c)3 (a + b + c) ≥ 32(ac − bc)2 .
Note that using a ≥ 0, again we have
3
a + 2(b + c)3 (a + b + c) ≥ 2(b + c)4 ≥ 32b2 c2 ≥ 32(bc − ca)2 .
4
Hence we are done. Equality holds if and only if a = 0; b = c or b = 0; a = c.
Remark
We also have the following results given c(b − a) > 0; a, b, c ≥ 0:
a3 b3 c3 16(a + b)3 (b + c)3 (c + a)3
• 3
+ 3
+ 3
+ ≥4
(b + c) (c + a) (a + b) (bc − ca)(b2 + c2 )(a + b + c)5
a3 b3 c3 8(a + b)3 (b + c)5 (c + a)3
• 3
+ 3
+ 3
+ p ≥ 4.
(b + c) (c + a) (a + b) bc(b2 + c2 )(a + b + c)5 2(bc − ca)(b2 + c2 )